Dirichlet's Theorem

Revision as of 12:19, 30 May 2019 by Hashtagmath (talk | contribs) (Stronger Result)
(diff) ← Older revision | Latest revision (diff) | Newer revision → (diff)

Theorem

For any positive integers $a$ and $m$ such that $(a,m)=1$, there exists infinitely many prime $p$ such that $p\equiv a\mod m$

Hence, for any arithmetic progression, unless it obviously contains finitely many primes (first term and common difference not coprime), it contains infinitely many primes.

Stronger Result

For any positive integers $a$ and $m$ such that $(a,m)=1$, \[\sum_{\substack{p\leq x\\ p\equiv a\mod m}}\frac{1}{p}=\frac{1}{\phi(m)}\log\log x+O(1)\] where the sum is over all primes $p$ less than $x$ that are congruent to $a$ mod $m$, and $\phi(x)$ is the totient function.

See Also